LSAC responded to my claim that a question was flawed. Forum

Prepare for the LSAT or discuss it with others in this forum.
User avatar
guano

Gold
Posts: 2264
Joined: Mon Feb 18, 2013 9:49 am

Re: LSAC responded to my claim that a question was flawed.

Post by guano » Fri Jun 07, 2013 2:43 pm

bp shinners wrote:
guano wrote:I don't actually see the flaw in the question. Please explain it to me like I were a 5 year old child
The problem is that the stimulus says, "Independent arbitration would avert a strike, but only if..." That's creating a world where something is necessary for independent arbitration to avert the strike, but it's not creating a world where nothing else can avert the strike.

It's like saying, "The Joker is planning to terrorize Gotham City. Batman would avert this rein of terror, but only if he doesn't have a swanky ball to attend. However, based on past experience, he will have a swanky ball to attend, so the Joker's rein of terror is likely."

The issue is that there are other heroes in the DC universe that can take out the Joker. Superman, Wonder Woman, Green Lantern, Green Arrow, even the Martian Manhunter; they could all stop his rein of terror. While you could make an argument that they might not be around, the conclusion should be that the Joker's rein of terror is possible, as it's not necessarily likely.

In short, I'm not buying the explanation of this as valid. They seem to hedge their bets even in the letter, saying that the reasoning isn't completely parallel. So I guess they're fine with shifting "Parallel" questions to "soft Parallel" questions, like "Must be True" vs. "Most Strongly Supported." Or they just messed up here and don't want to admit it. We'll have to see where this trend goes.
Your explanation is adding in conditionals that are not in the question

bp shinners

Gold
Posts: 3086
Joined: Wed Mar 16, 2011 7:05 pm

Re: LSAC responded to my claim that a question was flawed.

Post by bp shinners » Fri Jun 07, 2013 3:07 pm

guano wrote:
bp shinners wrote:
guano wrote:I don't actually see the flaw in the question. Please explain it to me like I were a 5 year old child
The problem is that the stimulus says, "Independent arbitration would avert a strike, but only if..." That's creating a world where something is necessary for independent arbitration to avert the strike, but it's not creating a world where nothing else can avert the strike.

It's like saying, "The Joker is planning to terrorize Gotham City. Batman would avert this rein of terror, but only if he doesn't have a swanky ball to attend. However, based on past experience, he will have a swanky ball to attend, so the Joker's rein of terror is likely."

The issue is that there are other heroes in the DC universe that can take out the Joker. Superman, Wonder Woman, Green Lantern, Green Arrow, even the Martian Manhunter; they could all stop his rein of terror. While you could make an argument that they might not be around, the conclusion should be that the Joker's rein of terror is possible, as it's not necessarily likely.

In short, I'm not buying the explanation of this as valid. They seem to hedge their bets even in the letter, saying that the reasoning isn't completely parallel. So I guess they're fine with shifting "Parallel" questions to "soft Parallel" questions, like "Must be True" vs. "Most Strongly Supported." Or they just messed up here and don't want to admit it. We'll have to see where this trend goes.
Your explanation is adding in conditionals that are not in the question
Can you point out where? I'm not seeing it. The only discrepancy I can see is that I said he will have a swanky ball to attend, and the original said that it's quite unlikely that they would agree to the terms. That doesn't negate my point about other heroes, however, and I'd stick by my analysis even with the switch in logical force.

User avatar
guano

Gold
Posts: 2264
Joined: Mon Feb 18, 2013 9:49 am

Re: LSAC responded to my claim that a question was flawed.

Post by guano » Fri Jun 07, 2013 3:13 pm

You're adding in extra possibilities, such as some other superhero coming in and saving the day

I've been doing logic games for a long time, and you really need to learn to think in the box. It's a closed world assignment

User avatar
Dr. Dre

Gold
Posts: 2337
Joined: Mon Feb 18, 2013 7:10 pm

Re: LSAC responded to my claim that a question was flawed.

Post by Dr. Dre » Fri Jun 07, 2013 3:14 pm

bp shinners can you diagram the stimulus with just a few variables so I can get a stronger sense of what you're getting at.

bp shinners

Gold
Posts: 3086
Joined: Wed Mar 16, 2011 7:05 pm

Re: LSAC responded to my claim that a question was flawed.

Post by bp shinners » Fri Jun 07, 2013 3:27 pm

guano wrote:You're adding in extra possibilities, such as some other superhero coming in and saving the day

I've been doing logic games for a long time, and you really need to learn to think in the box. It's a closed world assignment
This isn't a logic game, this is a logical reasoning question. In those, you most definitely can bring in other possibilities if the stimulus doesn't rule them out. I can point out dozens of questions where possibilities not mentioned in the stimulus show up in correct answer choices, especially in flaw/strengthen/weaken.

Now if the stimulus had said, "Only independent arbitration would avert a strike, but only if...", then I'd be with you. But without explicitly stating that independent arbitration is our only hope, then even Obi-Wan could come in and save the day.

Want to continue reading?

Register now to search topics and post comments!

Absolutely FREE!


Daily_Double

Silver
Posts: 1031
Joined: Tue Dec 04, 2012 8:45 pm

Re: LSAC responded to my claim that a question was flawed.

Post by Daily_Double » Fri Jun 07, 2013 3:32 pm

guano wrote:You're adding in extra possibilities, such as some other superhero coming in and saving the day

I've been doing logic games for a long time, and you really need to learn to think in the box. It's a closed world assignment
If I'm not mistaken, then your analysis of this question considers things sufficient to achieve a certain end as extra possibilities. Also, lecturing Shinners on how to think in this case seems at least unnecessary.

Let's apply your logic to the a post I made in the other thread:
Daily_Double wrote:I think the flaw would really be takes the absence of a condition which would make an action unlikely, to infer that the action would be likely. Or, alternatively, assumes, without providing justification, that there do not exist other means to prevent a strike which are likely. Alternatively, the argument fails to take into account other means which are both likely and would prevent a strike. But this is really just rewording your view of the flaw, since we can also interpret the stimulus into a separate conditional relationships:

Arbit ---> ~Strike
Abit would prevent strike ---> Agreement
~Agreement

Now the tricky part. What we can infer here is that arbitration would not prevent a strike. The argument infers from this that a strike is probable. So the argument takes the absence of a condition which would not result in an action to infer the presence of the action.
You would claim, or so it would appear, that any other method sufficient to prevent the strike is an extra possibility, when in fact questions using this logic appear often, though they're usually in absolute terms, as flaw questions. The only reason I could think of to support your claim that these possibilities are irrelevant is related to LSAC's argument that if a journalist makes a claim in the context of journalism, then that claim represents a relatively complete picture of the known facts. So how do your support your claim?

User avatar
Dr. Dre

Gold
Posts: 2337
Joined: Mon Feb 18, 2013 7:10 pm

Re: LSAC responded to my claim that a question was flawed.

Post by Dr. Dre » Fri Jun 07, 2013 3:32 pm

I still don't get what's so problematic. The conclusion is probabilistic.

It would have been different had it said that "the strike will happen." Or "the strike will occur."

bp shinners

Gold
Posts: 3086
Joined: Wed Mar 16, 2011 7:05 pm

Re: LSAC responded to my claim that a question was flawed.

Post by bp shinners » Fri Jun 07, 2013 3:33 pm

Dr. Dre wrote:bp shinners can you diagram the stimulus with just a few variables so I can get a stronger sense of what you're getting at.
Sure:
IAAS->BSAB
~BSAB
_________________
S

So if independent arbitration averts the strike, then both sides agreed it's binding.
Both sides don't agree it's binding.
So the valid conclusion is that independent arbitration won't avert the strike, not that a strike isn't likely. You need to add in that independent arbitration is the only thing that will avert the strike (or possibly even the only thing that's likely to avert the strike, I'd have to think on that).

I switched off the logical force here because we're arguing about an exclusivity fallacy; putting it back in, for me, doesn't change the sufficient/necessary fallacy. And that first term is a little weird because the original skirts the line between conditional and absolute statements (you can phrase it conditionally, but it's really absolute, which is why I represented it like that instead of as a conditional itself).

But it all comes down to the fact that we've said it's unlikely for independent arbitration to avert the strike, but we have in no way limited other options to avert the strike.

bp shinners

Gold
Posts: 3086
Joined: Wed Mar 16, 2011 7:05 pm

Re: LSAC responded to my claim that a question was flawed.

Post by bp shinners » Fri Jun 07, 2013 3:36 pm

Dr. Dre wrote:I still don't get what's so problematic. The conclusion is probabilistic.

It would have been different had it said that "the strike will happen." Or "the strike will occur."
The problem is the conclusion denotes strong probability - it is "likely", which is another word for "probable." Just because I strongly hint that we can't use one method to resolve the strike doesn't mean that it's unlikely for the strike to occur. It would be logically consistent for the reporter to throw out that last sentence about the strike happening and replace it with, "But luckily the government intervenes in these situations to prevent a strike, so the strike isn't going to happen." Without ruling out that as a possibility by saying that independent arbitration is our only (likely) solution, you leave the door open.

Want to continue reading?

Register for access!

Did I mention it was FREE ?


User avatar
guano

Gold
Posts: 2264
Joined: Mon Feb 18, 2013 9:49 am

Re: LSAC responded to my claim that a question was flawed.

Post by guano » Fri Jun 07, 2013 4:00 pm

bp shinners wrote:
guano wrote:You're adding in extra possibilities, such as some other superhero coming in and saving the day

I've been doing logic games for a long time, and you really need to learn to think in the box. It's a closed world assignment
This isn't a logic game, this is a logical reasoning question. In those, you most definitely can bring in other possibilities if the stimulus doesn't rule them out. I can point out dozens of questions where possibilities not mentioned in the stimulus show up in correct answer choices, especially in flaw/strengthen/weaken.

Now if the stimulus had said, "Only independent arbitration would avert a strike, but only if...", then I'd be with you. But without explicitly stating that independent arbitration is our only hope, then even Obi-Wan could come in and save the day.
The question is which of the following exhibits a pattern of similar reasoning.

bp shinners

Gold
Posts: 3086
Joined: Wed Mar 16, 2011 7:05 pm

Re: LSAC responded to my claim that a question was flawed.

Post by bp shinners » Fri Jun 07, 2013 4:06 pm

guano wrote:
bp shinners wrote:
guano wrote:You're adding in extra possibilities, such as some other superhero coming in and saving the day

I've been doing logic games for a long time, and you really need to learn to think in the box. It's a closed world assignment
This isn't a logic game, this is a logical reasoning question. In those, you most definitely can bring in other possibilities if the stimulus doesn't rule them out. I can point out dozens of questions where possibilities not mentioned in the stimulus show up in correct answer choices, especially in flaw/strengthen/weaken.

Now if the stimulus had said, "Only independent arbitration would avert a strike, but only if...", then I'd be with you. But without explicitly stating that independent arbitration is our only hope, then even Obi-Wan could come in and save the day.
The question is which of the following exhibits a pattern of similar reasoning.
I'm not dealing with the overall question; I'm dealing with the contention that the stimulus is valid. I don't believe it is. That creates issues because if the answer choice is valid but the stimulus isn't, then we've moved past our normal Parallel questions and the LSAC is now putting Most Parallel questions on there. Which is a big deal, especially if they start letting validity not be a criterion for an answer choice.

User avatar
guano

Gold
Posts: 2264
Joined: Mon Feb 18, 2013 9:49 am

Re: LSAC responded to my claim that a question was flawed.

Post by guano » Fri Jun 07, 2013 4:11 pm

bp shinners wrote:
guano wrote:
bp shinners wrote:
guano wrote:You're adding in extra possibilities, such as some other superhero coming in and saving the day

I've been doing logic games for a long time, and you really need to learn to think in the box. It's a closed world assignment
This isn't a logic game, this is a logical reasoning question. In those, you most definitely can bring in other possibilities if the stimulus doesn't rule them out. I can point out dozens of questions where possibilities not mentioned in the stimulus show up in correct answer choices, especially in flaw/strengthen/weaken.

Now if the stimulus had said, "Only independent arbitration would avert a strike, but only if...", then I'd be with you. But without explicitly stating that independent arbitration is our only hope, then even Obi-Wan could come in and save the day.
The question is which of the following exhibits a pattern of similar reasoning.
I'm not dealing with the overall question; I'm dealing with the contention that the stimulus is valid. I don't believe it is. That creates issues because if the answer choice is valid but the stimulus isn't, then we've moved past our normal Parallel questions and the LSAC is now putting Most Parallel questions on there. Which is a big deal, especially if they start letting validity not be a criterion for an answer choice.
So, you're basically creating an issue where there isn't one
Got it

bp shinners

Gold
Posts: 3086
Joined: Wed Mar 16, 2011 7:05 pm

Re: LSAC responded to my claim that a question was flawed.

Post by bp shinners » Fri Jun 07, 2013 4:32 pm

guano wrote: So, you're basically creating an issue where there isn't one
Got it
Nope, I'm trying to see if there's been a fundamental change in the way Parallel questions work. That's completely relevant, because if we have to start looking for different things in those answer choices, it's important to know it.

Register now!

Resources to assist law school applicants, students & graduates.

It's still FREE!


Daily_Double

Silver
Posts: 1031
Joined: Tue Dec 04, 2012 8:45 pm

Re: LSAC responded to my claim that a question was flawed.

Post by Daily_Double » Fri Jun 07, 2013 5:18 pm

Graeme, I completely agree that LSAC has a good argument, if we accept the truth of their supporting statements. But since it would be erroneous to say that a valid argument, though similar in structure, follows the same line of reasoning as a flawed argument, and it would appear that you share some of my issues, most notably the issue of whether or not the context of journalism entails a complete picture of the relevant facts, I would suppose that you are not completely satisfied with their explanation.

I think there are issues that as of yet, are unresolved. So my question is, are you going to reply with a counterargument to LSAC's response? I think this issue is above my pay grade, though if I ever pick up the part time job of tutoring this test, I'll come back to it. But thanks for posting it.

User avatar
Dr. Dre

Gold
Posts: 2337
Joined: Mon Feb 18, 2013 7:10 pm

Re: LSAC responded to my claim that a question was flawed.

Post by Dr. Dre » Fri Jun 07, 2013 5:49 pm

bp shinners:

if there was a flaw, would this question be placed in the "Flaws(But Not)" section of Lesson 14?

:D

bp shinners

Gold
Posts: 3086
Joined: Wed Mar 16, 2011 7:05 pm

Re: LSAC responded to my claim that a question was flawed.

Post by bp shinners » Fri Jun 07, 2013 5:52 pm

Dr. Dre wrote:bp shinners:

if there was a flaw, would this question be placed in the "Flaws(But Not)" section of Lesson 14?

:D
Nope. Because there is a flaw, which would mean that it is in that section. But it's not, so your conditional doesn't hold true ;-)

Now, this one COULD appear in that section.

User avatar
Dr. Dre

Gold
Posts: 2337
Joined: Mon Feb 18, 2013 7:10 pm

Re: LSAC responded to my claim that a question was flawed.

Post by Dr. Dre » Fri Jun 07, 2013 10:58 pm

damn :shock:

you're really good, bp shinners

Get unlimited access to all forums and topics

Register now!

I'm pretty sure I told you it's FREE...


User avatar
guano

Gold
Posts: 2264
Joined: Mon Feb 18, 2013 9:49 am

Re: LSAC responded to my claim that a question was flawed.

Post by guano » Fri Jun 07, 2013 11:30 pm

There is no flaw

User avatar
Dr. Dre

Gold
Posts: 2337
Joined: Mon Feb 18, 2013 7:10 pm

Re: LSAC responded to my claim that a question was flawed.

Post by Dr. Dre » Fri Jun 07, 2013 11:31 pm

guano wrote:There is no flaw
yes i know

WilliamDeWrites

New
Posts: 12
Joined: Mon Jul 27, 2009 1:24 pm

Re: LSAC responded to my claim that a question was flawed.

Post by WilliamDeWrites » Mon Jun 10, 2013 1:04 pm

I am with LSAC on this. I see their argument this way. Since a strike is being planned and some positive action must be done to avert it, a strike is likely. The “default” position is that a strike is likely. Then the argument says that arbitration would avert the strike, but that arbitration is unlikely. So, since what we know would avert the strike, arbitration, is unlikely, we revert back to the default: a strike is likely.

Some of you are making too much of the discussion of the journalist. All that does is point out that we have no good reason to think that anything other than arbitration will avert the strike, even if that is theoretically possible. So, in the context of this question, we have no good reason to doubt that a strike is likely. Good journalists want to get at the facts and this journalist is concerned about whether a strike is likely, so if there were good reason to think that something other than arbitration was likely to avert the strike, the journalist would most likely mention it. So again, in the context of the argument, we don’t have any good reason to think that it is very likely that anything else would avert the strike. Also, as Graeme pointed out, to weaken the conclusion that a strike is likely it is not enough to point out the possibility of something averting the strike. Something can be likely even if there’s a possibility that it might not happen (or be averted). To weaken this argument in the face of the reasons given you must establish the likelihood that something else will avert the strike. And there is nothing in the reasoning of the journalist’s argument or the context to establish that.

Also, speaking about flawed reasoning, it seems that some of you are way too quick to overgeneralize about LSAT reasoning, or parallel reasoning questions, or flaw questions, or trends on the LSAT based on a single question.

bp shinners

Gold
Posts: 3086
Joined: Wed Mar 16, 2011 7:05 pm

Re: LSAC responded to my claim that a question was flawed.

Post by bp shinners » Mon Jun 10, 2013 2:20 pm

WilliamDeWrites wrote:Good journalists want to get at the facts and this journalist is concerned about whether a strike is likely, so if there were good reason to think that something other than arbitration was likely to avert the strike, the journalist would most likely mention it. So again, in the context of the argument, we don’t have any good reason to think that it is very likely that anything else would avert the strike.
Alright, so what if it was a concerned citizen instead? A government expert on strikes? A member of the striking group? My problem with this line of argumentation is that I don't know when the LSAT views someone as having and presenting all the facts, and when someone doesn't. It's easy to say "Yes" for a journalist, but it doesn't help when the question changes slightly. That uncertainty makes me...uneasy.
Also, as Graeme pointed out, to weaken the conclusion that a strike is likely it is not enough to point out the possibility of something averting the strike. Something can be likely even if there’s a possibility that it might not happen (or be averted). To weaken this argument in the face of the reasons given you must establish the likelihood that something else will avert the strike. And there is nothing in the reasoning of the journalist’s argument or the context to establish that.
But when dealing with possibility/probability on the LSAT, you usually require certainty of the logical force. While leaving open the possibility of another action averting the strike doesn't mean that averting a strike is likely, it also doesn't mean that not averting the strike is likely. Without knowing anything about it, I'm not sure either way on the likelihood.
Also, speaking about flawed reasoning, it seems that some of you are way too quick to overgeneralize about LSAT reasoning, or parallel reasoning questions, or flaw questions, or trends on the LSAT based on a single question.
I'm more responding to the language used in the response. Parallel questions, traditionally, have a very high burden for correct answers, to the point where a single element of un-parallel reasoning can be enough to invalidate an answer. In their response, the LSAC concedes that, for this question, not every element of the stimulus is reflected in the answer choice. If that's the way they feel about Parallel questions, that's something that's important to know, as all of the test prep companies of whose methods I'm aware treat a correct answer choice as having all elements reflected.

Communicate now with those who not only know what a legal education is, but can offer you worthy advice and commentary as you complete the three most educational, yet challenging years of your law related post graduate life.

Register now, it's still FREE!


WilliamDeWrites

New
Posts: 12
Joined: Mon Jul 27, 2009 1:24 pm

Re: LSAC responded to my claim that a question was flawed.

Post by WilliamDeWrites » Mon Jun 10, 2013 2:57 pm

For BP Shinners:

In response to your first point, I don't think it is so important that it is a journalist in particular, just that it be someone unlikely to be ignoring germane evidence. And the evidence needed to weaken the argument is not that there be a possibility that something else might avert the strike (which there is), but there be a likelihood that something else avert the strike. And there is just no reason to think the latter. So, all things being equal, the argument is not flawed.

In response to your second point: You seem to be missing LSAC's point that in this argument the likelihood of the strike is not just inferred from the fact that arbitration is unlikely, but also from the fact that a strike is planned. When you've got a hammer everything looks like a nail. Just because the stuff about necessary and sufficient conditions is useful doesn't mean everything will fit neatly into that pattern. You've got to pay attention to exactly what the argument says. Like I said, when you rule out the sufficient condition, you revert to the default established by the planning and the need to avert, which is that a strike is likely.

In response to your third part: LSAC always hedges it bets by saying "most similar". Hardly any, if any, arguments are "exactly similar" in their pattern of reasoning. And the difference here is just that one argument has more steps than the other, not that there is in difference in the relevant steps. Seems like an easy call to me.

User avatar
guano

Gold
Posts: 2264
Joined: Mon Feb 18, 2013 9:49 am

Re: LSAC responded to my claim that a question was flawed.

Post by guano » Mon Jun 10, 2013 3:14 pm

WilliamDeWrites wrote:For BP Shinners:

In response to your first point, I don't think it is so important that it is a journalist in particular, just that it be someone unlikely to be ignoring germane evidence. And the evidence needed to weaken the argument is not that there be a possibility that something else might avert the strike (which there is), but there be a likelihood that something else avert the strike. And there is just no reason to think the latter. So, all things being equal, the argument is not flawed.

In response to your second point: You seem to be missing LSAC's point that in this argument the likelihood of the strike is not just inferred from the fact that arbitration is unlikely, but also from the fact that a strike is planned. When you've got a hammer everything looks like a nail. Just because the stuff about necessary and sufficient conditions is useful doesn't mean everything will fit neatly into that pattern. You've got to pay attention to exactly what the argument says. Like I said, when you rule out the sufficient condition, you revert to the default established by the planning and the need to avert, which is that a strike is likely.

In response to your third part: LSAC always hedges it bets by saying "most similar". Hardly any, if any, arguments are "exactly similar" in their pattern of reasoning. And the difference here is just that one argument has more steps than the other, not that there is in difference in the relevant steps. Seems like an easy call to me.
But, but, but,
LSAC wrong, LSAC wrong

09042014

Diamond
Posts: 18203
Joined: Wed Oct 14, 2009 10:47 pm

Re: LSAC responded to my claim that a question was flawed.

Post by 09042014 » Mon Jun 10, 2013 3:37 pm

WilliamDeWrites wrote:For BP Shinners:

In response to your first point, I don't think it is so important that it is a journalist in particular, just that it be someone unlikely to be ignoring germane evidence. And the evidence needed to weaken the argument is not that there be a possibility that something else might avert the strike (which there is), but there be a likelihood that something else avert the strike. And there is just no reason to think the latter. So, all things being equal, the argument is not flawed.

In response to your second point: You seem to be missing LSAC's point that in this argument the likelihood of the strike is not just inferred from the fact that arbitration is unlikely, but also from the fact that a strike is planned. When you've got a hammer everything looks like a nail. Just because the stuff about necessary and sufficient conditions is useful doesn't mean everything will fit neatly into that pattern. You've got to pay attention to exactly what the argument says. Like I said, when you rule out the sufficient condition, you revert to the default established by the planning and the need to avert, which is that a strike is likely.

In response to your third part: LSAC always hedges it bets by saying "most similar". Hardly any, if any, arguments are "exactly similar" in their pattern of reasoning. And the difference here is just that one argument has more steps than the other, not that there is in difference in the relevant steps. Seems like an easy call to me.

Your first part is irrelevant because this questions isn't about flaws in reason but parallel reasoning and the answer doesn't require the same assumption that the person is giving all evidence. That right there makes it unparallel.

I disagree with your last point. PR questions are always exactly parallel. This one was just kinda parallel. It's a shitty question that could be easily fixed by adding "only." Maybe they can get off saying "we said most similar" but it's still a shitty question.

And several people have made a good point, that if LSAC is moving from their effective exactly standard, to actually relying on "most similar," it is important to know.

I'm guessing it's just a sloppy question.

bp shinners

Gold
Posts: 3086
Joined: Wed Mar 16, 2011 7:05 pm

Re: LSAC responded to my claim that a question was flawed.

Post by bp shinners » Mon Jun 10, 2013 3:46 pm

WilliamDeWrites wrote:For BP Shinners:

In response to your first point, I don't think it is so important that it is a journalist in particular, just that it be someone unlikely to be ignoring germane evidence.
Ignoring the other points (because DF hit some of them the same way I would, and others are things on which we just inherently disagree), which people are unlikely to ignore germane evidence, and why are journalists a part of this group? I'm not saying that I disagree with it the case of journalists (though I do). But without knowing whose view I can trust and whose view I can't, each time a question is framed in this way, I have a dilemma. Can I trust the city councilperson? The concerned citizen? The Mayor? Zach? People are going to disagree over groups that are trustworthy, and so it makes these questions rely on you having a viewpoint that aligns with that of the LSAC, not a viewpoint that aligns with logic.

Seriously? What are you waiting for?

Now there's a charge.
Just kidding ... it's still FREE!


Post Reply

Return to “LSAT Prep and Discussion Forum”